Is conservation of angular momentum a hoax?

In summary, the conversation discusses a problem involving a particle's motion in the xy plane and the conservation of angular momentum in an isolated system. The statement "If the net external torque acting on a system is zero, the angular momentum L of the system remains constant, no matter what changes take place within the system" is false, as shown by the equation m(r x v) ≠ m(r' x v). The incorrect assumption was made due to a misunderstanding of how to calculate the cross product of two vectors.
  • #1
Chozen Juan
14
0

Homework Statement


(This is a problem I myself created, so it may sound a bit trivial/stupid.) A particle of mass m in the xy plane has velocity v and a radius vector r with respect to some origin. After some time Δt, the same particle has velocity v and a radius vector r' with respect to the origin. Throughout the particle's motion, the (supposedly) isolated system consisting of this particle is not subject to any external force nor any external torque.

m(r x v) ≠ m(r' x v)
Li Lf

Clearly, angular momentum of the system about the origin is not conserved even though there is no net external torque on the system.

Is the following statement false? "If the net external torque acting on a system is zero, the angular momentum L of the system remains constant, no matter what changes take place within the system."

Attached is a figure of the problem.

Homework Equations



L [/B]= m(r x v)

The Attempt at a Solution



I'm stumped!
 

Attachments

  • Ang. momentum.JPG
    Ang. momentum.JPG
    37.1 KB · Views: 423
Last edited:
Physics news on Phys.org
  • #2
Chozen Juan said:
m(r x v) ≠ m(r' x v)

Why do you think the two expressions are not equal ?

Do you know how to calculate cross product of two vectors ? If not , please look up and your confusion will be resolved .
 
  • #3
conscience said:
Why do you think the two expressions are not equal ?

Do you know how to calculate cross product of two vectors ? If not , please look up and your confusion will be resolved .
SHOOT! I knew there was something stupid about this problem. I thought they weren't equal since the angles between the position and velocity vectors were different... but like an idiot, I forgot to take into account that the position vector changes magnitude as well. I have no idea how I missed that. This didn't even have to do with physics. It was purely a math problem.
 

Related to Is conservation of angular momentum a hoax?

1. What is angular momentum and why is it important?

Angular momentum is a physical quantity that describes the rotational motion of an object. It is important because it is a fundamental law of physics that governs the behavior of rotating systems and plays a crucial role in many natural phenomena, such as the motion of planets and the stability of spinning objects.

2. Is conservation of angular momentum a proven scientific concept?

Yes, the conservation of angular momentum is a well-established principle in physics that has been extensively tested and verified through numerous experiments and observations. It is an integral part of the laws of motion and has been confirmed by countless scientific studies.

3. Can angular momentum be created or destroyed?

No, according to the law of conservation of angular momentum, the total angular momentum of a system remains constant and cannot be created or destroyed. It can only be transferred between different parts of the system or converted into other forms of energy.

4. How does angular momentum relate to conservation of energy?

Angular momentum is closely related to the conservation of energy, as both are fundamental laws of physics that govern the behavior of physical systems. In fact, conservation of energy can be derived from the conservation of angular momentum, as any change in the rotational energy of a system is accompanied by a corresponding change in its angular momentum.

5. Are there any exceptions to the conservation of angular momentum?

While the conservation of angular momentum is a universal law, there are some situations where it may appear to be violated. For example, in quantum mechanics, subatomic particles can exhibit a phenomenon known as spin, which allows them to have an intrinsic angular momentum. However, this does not actually violate the conservation of angular momentum, as the total angular momentum of the system is still conserved.

Similar threads

  • Introductory Physics Homework Help
Replies
2
Views
851
  • Introductory Physics Homework Help
Replies
17
Views
222
  • Introductory Physics Homework Help
10
Replies
335
Views
8K
  • Introductory Physics Homework Help
Replies
5
Views
220
  • Introductory Physics Homework Help
Replies
10
Views
965
  • Introductory Physics Homework Help
Replies
30
Views
2K
  • Introductory Physics Homework Help
Replies
4
Views
510
  • Introductory Physics Homework Help
Replies
23
Views
966
  • Introductory Physics Homework Help
Replies
2
Views
1K
Replies
13
Views
990
Back
Top